✫¸.•°*”˜˜”*°•✫ Ṱђầภ Ḉђết ✫•°*”˜˜”*°•.¸✫

Giới thiệu về bản thân

....
0
0
0
0
0
0
0
(Thường được cập nhật sau 1 giờ!)

Bài làm.

Ta có: M là trung điểm AB => M nằm giữa A và B => MA = MB

=> AM + MB = AB => MB + MB = AB => 2MB = AB => MB = AB/2 (1)

Lại có: C là tia đối của BM => B nằm giữa C và M

=> MB + BC = MC => BC = MC - BM (2)

Ta lấy (1) + (2) => MB + BC = AB/2 + (MC - BM)

=> MC = AB/2 + BC

=> MC = AB/2 + 2*BC/2

=> MC = (AB + 2BC)/2 = (AB + BC + BC)/2 = [(AB + BC) + BC]/2 = (AC + BC)/2 

Vậy MC = (AC + BC)/2 (đpcm)

- Số chia cho $9$ dư $7$ là: \(9\times2+7=25\)

\(\Rightarrow\) \(25\div3=8\) dư $1$

Trường hợp $1$

\(\left\{{}\begin{matrix}3x-1=1\\2y+1=4\end{matrix}\right.\Rightarrow\left\{{}\begin{matrix}x=\dfrac{2}{3}\\y=\dfrac{3}{2}\end{matrix}\right.\left(\text{loại}\right)\)

Trường hợp $2$

\(\left\{{}\begin{matrix}3x-1=4\\2y+1=1\end{matrix}\right.\Rightarrow\left\{{}\begin{matrix}x=\dfrac{5}{3}\\y=0\end{matrix}\right.\left(\text{loại}\right)\)

Trường hợp $3$

\(\left\{{}\begin{matrix}3x-1=2\\2y+1=2\end{matrix}\right.\Rightarrow\left\{{}\begin{matrix}x=1\\y=\dfrac{1}{2}\end{matrix}\right.\left(\text{loại}\right)\)

\(\dfrac{2003\times1999-2003\times999}{2004\times999+1004}=\) \(\dfrac{2003\times\left(1999-999\right)}{\left(1000+1004\right)\times999+1004}\)

\(=\dfrac{2003\times1000}{1000\times999+1004\times999+1004}\)

\(=\dfrac{2003\times1000}{1000\times999+1004\times\left(999+1\right)}\)

\(=\dfrac{2003\times1000}{1000\times999+1004\times1000}\)

\(=\dfrac{2003\times1000}{1000\times\left(999+1004\right)}\)

\(=\dfrac{2003}{2003}=1\)

\(3^{4x+1}=27^{x+3}\)

\(\Rightarrow3^{4x+1}=3^{3\left(x+3\right)}\)

\(\Rightarrow4x+1=3\left(x+3\right)\)

\(\Rightarrow4x-3x=9+1\)

\(\Rightarrow x=10\)

- Diện tích của mảnh đất là:\(28\times15=420\left(m^2\right)\)

- Diện tích đất làm nhà là: \(25\%\times420=105\left(m^2\right)\)

Số học sinh nam là: \(40-45\%\times40=22\left(hs\right)\)

\(2364.37+2364.62+2364+100\)

\(=2364.\left(37+62+1\right)+100\)

\(=2364.100+100\)

\(=236400+100\)

\(=236500\)

\(0,8.96+1,6.2\)

\(=\dfrac{4}{5}.96+\dfrac{8}{5}.2\)

\(=\dfrac{4}{5}.96+\left(\dfrac{4}{5}+\dfrac{4}{5}\right).2\)

\(=\dfrac{4}{5}.96+\dfrac{4}{5}.2+\dfrac{4}{5}.2\)

\(=\dfrac{4}{5}.\left(96+2+2\right)\)

\(=\dfrac{4}{5}.100\)

\(=\dfrac{400}{5}\)

\(=80\)